Recent questions tagged goclasses2024-mockgate-13

903
views
1 answers
5 votes
A total of $n$ points are equally spaced around a circle and are labelled with the integers $1$ to $n$, in order. Two points are called diametrically opposite if the ... $ and $35$ are diametrically opposite, then $n$ equals$54$55$56$57$
459
views
1 answers
4 votes
In a factory, Erika assembles $3$ calculators in the same amount of time that Nick assembles $2$ calculators. Also, Nick assembles $1$ calculator in the same ... $ calculators?$30$24$27$33$
472
views
1 answers
1 votes
A small square is drawn inside a larger square as shown. The area of the shaded region and the area of the unshaded region are each $18 \mathrm{~cm}^2$. What is the side length of the larger square?$3$4$6$9$
420
views
1 answers
4 votes
Chris received a mark of $50 \%$ on a recent test. Chris answered $13$ of the first $20$ questions correctly. Chris also answered $25 \%$ of the remaining ... $38$32$24$
497
views
1 answers
4 votes
414
views
2 answers
2 votes
A positive integer $n$ is a multiple of $7.$ The square root of $n$ is between $17$ and $18.$ How many possible values of $n$ are there?$2$3$4$5$
379
views
2 answers
1 votes
Ben participates in a prize draw. He receives one prize that is equally likely to be worth $\$ 5, \$ 10$ or $\$ ... $ 50?$1 / 6$1 / 3$1 / 2$1 / 4$
774
views
1 answers
5 votes
Five balls, numbered $1$ to $5,$ are placed in order on a table. A sequence of steps is performed on the balls. In step $1,$ the rightmost ball is picked ... . Which of the following is a possible value of $\mathrm{N}?$2023$2024$2025$2026$
398
views
1 answers
3 votes
Reading from left to right, the sequence consists of $6\;\text{X's},$ followed by $24\; \text{Y's},$ followed by $96\;\text{X's}.$ Hence, there are a total ... $n$. Which of the following is one of the possible values of $n?$72$74$76$80$
410
views
0 answers
2 votes
The significance of the Magna Carta lies not in its ________ provisions, but in its broader impact: it made the king subject to the law.specificrevolutionaryimplicitcontroversial
1.0k
views
2 answers
7 votes
Let $f(x)$ be a real-valued function all of whose derivatives exist. Recall that a point $x_0$ in the domain is called an inflection point of $f(x)$ ... and $x_0=6$, both are inflection points.The function does not have an inflection point.
572
views
1 answers
6 votes
Let $x$ be a random variable possessing the probability density function$f(x)= \begin{cases}c x & , x \in[0,10] \\ 0 & , \text { otherwise }\end{cases}$ ... $\dfrac{1}{100}$ $\dfrac{3}{100}$ $\dfrac{5}{100}$ $\dfrac{7}{100}$
635
views
1 answers
8 votes
If $A$ is a $3 \times 3$ matrix such that $A\left(\begin{array}{l}0 \\ 1 \\ 2\end{array}\right)=\left(\begin{array}{l}1 \\ 0 \\ 0\end{array}\right)$ ... $\left(\begin{array}{r}9 \\ 10 \\ 11\end{array}\right)$
352
views
1 answers
4 votes
Given that $f(n)=O(g(n))$ (where $\mathrm{O}$ is Big-O) and $f(n)=\Omega(g(n))$, which of the following statement is always true?$f(n)=o(g(n))$ (here $o$ is small-$o).$f(n)=\theta(g(n))$.$f(n)=\omega(g(n))$.Both A and B are always true.
889
views
1 answers
4 votes
Professor Fiorina uses the following algorithm to merge $k$ sorted lists, each containing $n / k$ elements.She takes the first list and merges it with the second list using a ... $\theta(k \log n)$
568
views
1 answers
6 votes
Suppose we constructed the binary search tree shown by starting with an empty tree and inserting one element at a time from an input sequence, without any rotations or other ... $3$ came before $14$ and $16$ came before $28.$
624
views
1 answers
3 votes
Suppose that a binary min-heap stores six elements with priorities $10,20,30,40,50$, and $60$ in its array $\text{A}.$ What is the largest of these items that could be stored in $\text{A}[2]?$ (indexing starts from zero)
475
views
1 answers
5 votes
What will be output printed by the following program?#include<stdio.h> main() { int c=4; switch(c) { c=c-1; case 4: ... $\text{IITB IISc IITM}$\text{IITB}$\text{IITB IITM IITD}$
384
views
0 answers
4 votes
What will be the output of the following program?#include <stdio.h> int main() { int a[2][2] = { {1, 2}, {3, 4} }; int (*p)[2][2]; p = &a; printf("%d", (*p)[0][1]); return 0; }$1$2$3$4$
685
views
1 answers
5 votes
Suppose we have a multi-level index file system, and this file system has $512$ Bytes blocks, inodes with $3$ direct, $1$ single-indirect, $2$ double- ... single-indirect block number be used?$1536$ Bytes$1537$ Bytes$512$ Bytes$1500$ Bytes
525
views
1 answers
5 votes
Consider a virtual memory system that uses paging. Virtual and physical addresses are both $32$ ... {0x00875b65}$\textsf{0x00875}$Translation is not possible as per the given page table
514
views
1 answers
4 votes
An IP router implementing Classless Inter-domain Routing (CIDR) receives a packet with address $128.96.39.10.$ ... $\text{R}3$\text{R}4$\text{R}6$
715
views
1 answers
6 votes
Which of the following statements is/are false?If a context-free grammar $\mathrm{G}$ is in Chomsky's normal form, then $\mathrm{G}$ ... $100$-state DFA that accepts $\mathrm{L}$.
627
views
1 answers
6 votes
Let $\mathrm{M}$ be a $8$-state Deterministic Finite Automaton over the alphabet $\{a, b, c\}$. If the language accepted by $\text{M}$ i.e. $\text{L(M)}$ is finite, then the maximum possible cardinality of $\text{L(M)}$ is __________.
439
views
1 answers
4 votes
A garage door opens if it ever sees the password $011$ in a transmission. More formally, this FSM takes a bitstring consisting of $\text{0's}$ and $\text{1's}$ as its input, and ... $3 - (1/0)$Arrow $4 - (1/0)$Arrow $5 - (1/1)$
458
views
1 answers
4 votes
The figure below represents the Karnaugh map for a function $\text{F(A,B,C,D).}$ Note, $\text{ X'}$ stands for don't care.The simplified ... can be converted into a circuit implementation using only NAND gates, which is shown in:abcd
479
views
2 answers
3 votes
What statement is correct for $f(A, B)$ in the following circuit?$f(A, B)=\overline{\overline{A \cdot B} \cdot(A+B)}$ when Control $=1$ ... }$ when Control $=1$f(A, B)=\overline{A} \cdot \overline{B}$ when Control $=0$
408
views
1 answers
3 votes
A group $G$ in which $(a b)^2=a^2 b^2$ for all $a, b$ in $G$ is necessarilyfinitecyclicabeliannone of the above
594
views
1 answers
5 votes
In two's complement, what is the minimum number of bits needed to represent the numbers $-1$ and the number $1$ respectively?$1$ and $2$2$ and $2$2$ and $1$1$ and $1$
473
views
1 answers
3 votes
Let $\text{R}_1, \text{R}_2, \ldots,\text{ }R_n$ be a decomposition of schema $\text{U}$. Let $u(\text{U})$ ... \ldots r_n$u=r_1 \bowtie r_2 \bowtie r_3 \bowtie r_4 \bowtie . . r_n$None of the above